1950 AHSME Problems/Problem 39

Revision as of 13:40, 18 January 2012 by Mrdavid445 (talk | contribs) (Created page with "==Problem== Given the series <math> 2\plus{}1\plus{}\frac {1}{2}\plus{}\frac {1}{4}\plus{}...</math> and the following five statements: (1) the sum increases without limit (2) t...")
(diff) ← Older revision | Latest revision (diff) | Newer revision → (diff)

Problem

Given the series $2\plus{}1\plus{}\frac {1}{2}\plus{}\frac {1}{4}\plus{}...$ (Error compiling LaTeX. Unknown error_msg) and the following five statements: (1) the sum increases without limit (2) the sum decreases without limit (3) the difference between any term of the sequence and zero can be made less than any positive quantity no matter how small (4) the difference between the sum and 4 can be made less than any positive quantity no matter how small (5) the sum approaches a limit Of these statments, the correct ones are:

$\textbf{(A)}\ \text{Only }3 \text{ and }4\qquad \textbf{(B)}\ \text{Only }5 \qquad \textbf{(C)}\ \text{Only }2\text{ and }4 \qquad \textbf{(D)}\ \text{Only }2,3\text{ and }4 \qquad \textbf{(E)}\ \text{Only }4\text{ and }5$